Finding the flux (Divergence Theorem)

Click For Summary
The discussion focuses on using the divergence theorem to find the flux of the vector field F = (y^3)i + (x^3)j + (3z^2)k out of the surface of the paraboloid defined by z = x^2 + y^2, z ≤ 9. The user attempts to set up the integral by considering slices of fixed z and integrating over a variable radius r, which depends on z rather than a constant value. They express the integral as a triple integral but seek clarification on how to integrate with variable limits for r and the surface integral involving Fn ds. The user acknowledges a mistake in their volume integral calculation but remains uncertain about the surface integral and requests suggestions for resolving their confusion. The conversation highlights the complexities of applying the divergence theorem in this context.
nb89
Messages
37
Reaction score
0

Homework Statement


By using divergence theorem find the flux of vector F out of the surface of the paraboloid z = x^2 + y^2, z<=9, when F = (y^3)i + (x^3)j + (3z^2)k

Homework Equations


Divergence theorem equation stated in the attempt part

The Attempt at a Solution


2qa2ul3.jpg
 
Physics news on Phys.org
Suppose you are doing the z-integral first, so we are looking at a slice of fixed z. Then the slice looks like a disk of radius rmax. Instead of 0 to 3, you want to integrate each slice over r from 0 to rmax. You can express rmax in terms of z.
So you will get
\int_0^9 \int_0^{r_\mathrm{max}(z)} \int_0^{2\pi} 6 z r \, \mathrm{d}\varphi \, \mathrm{d}r \, \mathrm d{z}
where rmax(z) depends on z instead of being identically equal to 3 as you have now.
 
Last edited:
CompuChip said:
Suppose you are doing the z-integral first, so we are looking at a slice of fixed z. Then the slice looks like a disk of radius rmax. Instead of 0 to 3, you want to integrate each slice over r from 0 to rmax. You can express rmax in terms of z.
So you will get
\int_0^9 \int_0^{r_\mathrm{max}(z)} \int_0^{2\pi} 6 z r \, \mathrm{d}\varphi \, \mathrm{d}r \, \mathrm d{z}
where rmax(z) depends on z instead of being identically equal to 3 as you have now.
I understand what you mean about the radius changing with z, but how would i integrate dr with limits rmax(z)?
Also what about the Fn ds double integral?
 
For some given z, what is the maximum radius (i.e. the upper boundary of your r-integration)?

I haven't looked into the other approach (where you first apply the divergence theorem) but it does look a bit more complicated to me (finding the correct normal unit vector and all).
 
Just thought id bump this question. I understand my error when calculating the volume integral, the problem I'm left with is the surface integral. I still can't spot the mistake there. Any suggestions? Thanks.
 
Question: A clock's minute hand has length 4 and its hour hand has length 3. What is the distance between the tips at the moment when it is increasing most rapidly?(Putnam Exam Question) Answer: Making assumption that both the hands moves at constant angular velocities, the answer is ## \sqrt{7} .## But don't you think this assumption is somewhat doubtful and wrong?

Similar threads

  • · Replies 1 ·
Replies
1
Views
1K
Replies
6
Views
2K
Replies
8
Views
3K
  • · Replies 10 ·
Replies
10
Views
1K
  • · Replies 6 ·
Replies
6
Views
2K
  • · Replies 15 ·
Replies
15
Views
2K
  • · Replies 6 ·
Replies
6
Views
2K
  • · Replies 6 ·
Replies
6
Views
3K
Replies
2
Views
2K
Replies
9
Views
2K